Difference between revisions of "Mock AIME 2 2006-2007 Problems/Problem 9"

Line 1: Line 1:
 
== Problem ==
 
== Problem ==
In right triangle <math>ABC,</math> <math>\angle C=90^\circ.</math> Cevians <math>AX</math> and <math>BY</math> intersect at <math>P</math> and are drawn to <math>BC</math> and <math>AC</math> respectively such that <math>\frac{BX}{CX}=\frac23</math> and <math>\frac{AY}{CY}=\sqrt 3.</math> If <math>\tan \angle APB= \frac{a+b\sqrt{c}}{d},</math> where <math>a,b,</math> and <math>d</math> are relatively prime and <math>c</math> has no perfect square divisors excluding <math>1,</math> find <math>a+b+c+d.</math>
+
In right triangle <math>ABC,</math> <math>\angle C=90^\circ.</math> Cevians <math>AX</math> and <math>BY</math> intersect at <math>P</math> and are drawn to <math>BC</math> and <math>AC</math> respectively such that <math>\frac{BX}{CX}=\frac23</math>, <math>\frac{AY}{CY}=\sqrt 3,</math> and <math>CY=CX-BX</math>. If <math>\tan \angle APB= -\frac{a+b\sqrt{c}}{d},</math> where <math>a,b,</math> and <math>d</math> are relatively prime and <math>c</math> has no perfect square divisors excluding <math>1,</math> find <math>a+b+c+d.</math>
  
 
==Solution==
 
==Solution==
 
{{solution}}
 
{{solution}}
 +
<asy>
 +
import olympiad;
 +
size(200);
 +
defaultpen(linewidth(0.8));
 +
pair A=(0,6),B=(5,0),C=origin,X=(3,0),Y=A/(sqrt(3)+1);
 +
draw(A--B--C--A--X^^Y--B);
 +
label("$A$",A,N);
 +
label("$B$",B,E);
 +
label("$C$",C,SW);
 +
label("$X$",X,S);
 +
label("$Y$",Y,W);
 +
pair P=extension(A,X,B,Y);
 +
pair D=foot(P,B,C),E=foot(P,A,C);
 +
draw(D--P--E,linetype("4 4"));
 +
draw(rightanglemark(A,C,B,5)^^rightanglemark(A,E,P,5)^^rightanglemark(P,D,B,5));
 +
label("$D$",D,S);
 +
label("$E$",E,W);
 +
label("$P$",P,0.5(dir(P--B)+dir(P--A)));
 +
</asy>
 +
 +
Define <math>CX=3x</math>, <math>XB=2x</math>, <math>CY=y</math>, and <math>YA=y\sqrt3</math>.  Note that <math>CY=CX-BX</math> implies <math>y=3x-2x=x</math>.
 +
 +
Let <math>D</math> and <math>E</math> be the projections of <math>P</math> onto the legs <math>AC</math> and <math>BC</math> respectively.  Remark that <math>\tan\angle DPB=\tan \angle CXB=5</math> and <math>\tan\angle APE=\tan\angle AXC=1+\sqrt3</math>, so <cmath>\begin{align*}\tan(\angle DPB+\angle APE)=\dfrac{\tan\angle DPB+\tan\angle APE}{1-\tan\angle DPB\tan\angle APE}=\dfrac{5+1+\sqrt3}{1-5(1+\sqrt3)}=-\dfrac{6+\sqrt3}{4+5\sqrt3}.\end{align*}</cmath>  Since <math>\angle APB+(\angle APE+\angle DPB)=270^\circ</math>, we have <cmath>\begin{align*}\tan\angle APB&=\tan[270^\circ-(\angle APE+\angle BPD)]\\&=\cot (\angle APE+\angle BPD)\\&=-\dfrac{4+5\sqrt3}{6+\sqrt3}=-\dfrac{9+26\sqrt3}{33}.\end{align*}</cmath> The requested answer is thus <math>9+26+3+33=\boxed{071}.</math>
  
 
==See Also==
 
==See Also==
 
{{Mock AIME box|year=2006-2007|n=2|num-b=8|num-a=10}}
 
{{Mock AIME box|year=2006-2007|n=2|num-b=8|num-a=10}}

Revision as of 23:22, 3 May 2014

Problem

In right triangle $ABC,$ $\angle C=90^\circ.$ Cevians $AX$ and $BY$ intersect at $P$ and are drawn to $BC$ and $AC$ respectively such that $\frac{BX}{CX}=\frac23$, $\frac{AY}{CY}=\sqrt 3,$ and $CY=CX-BX$. If $\tan \angle APB= -\frac{a+b\sqrt{c}}{d},$ where $a,b,$ and $d$ are relatively prime and $c$ has no perfect square divisors excluding $1,$ find $a+b+c+d.$

Solution

This problem needs a solution. If you have a solution for it, please help us out by adding it. [asy] import olympiad; size(200); defaultpen(linewidth(0.8)); pair A=(0,6),B=(5,0),C=origin,X=(3,0),Y=A/(sqrt(3)+1); draw(A--B--C--A--X^^Y--B); label("$A$",A,N); label("$B$",B,E); label("$C$",C,SW); label("$X$",X,S); label("$Y$",Y,W); pair P=extension(A,X,B,Y); pair D=foot(P,B,C),E=foot(P,A,C); draw(D--P--E,linetype("4 4")); draw(rightanglemark(A,C,B,5)^^rightanglemark(A,E,P,5)^^rightanglemark(P,D,B,5)); label("$D$",D,S); label("$E$",E,W); label("$P$",P,0.5(dir(P--B)+dir(P--A))); [/asy]

Define $CX=3x$, $XB=2x$, $CY=y$, and $YA=y\sqrt3$. Note that $CY=CX-BX$ implies $y=3x-2x=x$.

Let $D$ and $E$ be the projections of $P$ onto the legs $AC$ and $BC$ respectively. Remark that $\tan\angle DPB=\tan \angle CXB=5$ and $\tan\angle APE=\tan\angle AXC=1+\sqrt3$, so \begin{align*}\tan(\angle DPB+\angle APE)=\dfrac{\tan\angle DPB+\tan\angle APE}{1-\tan\angle DPB\tan\angle APE}=\dfrac{5+1+\sqrt3}{1-5(1+\sqrt3)}=-\dfrac{6+\sqrt3}{4+5\sqrt3}.\end{align*} Since $\angle APB+(\angle APE+\angle DPB)=270^\circ$, we have \begin{align*}\tan\angle APB&=\tan[270^\circ-(\angle APE+\angle BPD)]\\&=\cot (\angle APE+\angle BPD)\\&=-\dfrac{4+5\sqrt3}{6+\sqrt3}=-\dfrac{9+26\sqrt3}{33}.\end{align*} The requested answer is thus $9+26+3+33=\boxed{071}.$

See Also

Mock AIME 2 2006-2007 (Problems, Source)
Preceded by
Problem 8
Followed by
Problem 10
1 2 3 4 5 6 7 8 9 10 11 12 13 14 15